Difference between revisions of "1990 AHSME Problems/Problem 29"
(→Solution 1) |
m (→Solution 1) |
||
Line 10: | Line 10: | ||
== Solution 1 == | == Solution 1 == | ||
− | Notice that inclusion of the integers between <math>34</math> to <math>100</math> inclusive is allowed as long as no integer between <math>11</math> and <math>33</math> inclusive is within the set. This provides a total of <math>100 - 34 + 1</math> = | + | Notice that inclusion of the integers between <math>34</math> to <math>100</math> inclusive is allowed as long as no integer between <math>11</math> and <math>33</math> inclusive is within the set. This provides a total of <math>100 - 34 + 1</math> = 67<math> solutions. |
− | Further analyzing the remaining integers between <math>1< | + | Further analyzing the remaining integers between </math>1<math> and </math>10<math>, we notice that we can include all the numbers except </math>3<math> (as including </math>3<math> would force us to remove both </math>9<math> and </math>1<math>) to obtain the maximum number of </math>9<math> solutions. |
− | Thus, 67 + 9 = 76 <math>\fbox{D} | + | Thus, </math>67 + 9 = 76<math>, yielding our answer, </math>\fbox{D}$ |
== Solution 2 == | == Solution 2 == |
Revision as of 13:45, 7 May 2017
Contents
Problem
A subset of the integers has the property that none of its members is 3 times another. What is the largest number of members such a subset can have?
Solution 1
Notice that inclusion of the integers between to inclusive is allowed as long as no integer between and inclusive is within the set. This provides a total of = 67$solutions.
Further analyzing the remaining integers between$ (Error compiling LaTeX. Unknown error_msg)11033919$solutions.
Thus,$ (Error compiling LaTeX. Unknown error_msg)67 + 9 = 76\fbox{D}$
Solution 2
Write down in a column the elements which are indivisible by three, and then follow each one by
We can take at most elements from the first row, and at most elements from each of the next seven rows. After that we can take only from any following row. Thus the answer is the number of integers between and inclusive which are indivisible by three.
There are multiples of three in that range, so there are non-multiples, and , which is
See also
1990 AHSME (Problems • Answer Key • Resources) | ||
Preceded by Problem 28 |
Followed by Problem 29 | |
1 • 2 • 3 • 4 • 5 • 6 • 7 • 8 • 9 • 10 • 11 • 12 • 13 • 14 • 15 • 16 • 17 • 18 • 19 • 20 • 21 • 22 • 23 • 24 • 25 • 26 • 27 • 28 • 29 • 30 | ||
All AHSME Problems and Solutions |
The problems on this page are copyrighted by the Mathematical Association of America's American Mathematics Competitions.